The Student Room Group

Uncertainty

Two resistors of resistances 120 ohms and 500 ohms are connected in parallel. The percentage uncertainty in the value of resistance of each resistor is 10%.

What is the correct value of the total resistance and the percentage uncertainty?
For resistors a and b in parallel is it given: 1/Rtotal = 1/Ra + 1/Rb

so Rtotal = 1/(1/120 + 1/500) = 3000/31 = 96.774... = 97 to 2.s.f

For uncertainty in different operations:

when adding/subtracting: add the absolute uncertainties
when multiplying/dividing: add the percentage uncertainties


so we were dividing so 10% + 10% = 20%

97ohms +or- 20%
Reply 2
Original post by old_survivor
For resistors a and b in parallel is it given: 1/Rtotal = 1/Ra + 1/Rb

so Rtotal = 1/(1/120 + 1/500) = 3000/31 = 96.774... = 97 to 2.s.f

For uncertainty in different operations:

when adding/subtracting: add the absolute uncertainties
when multiplying/dividing: add the percentage uncertainties


so we were dividing so 10% + 10% = 20%

97ohms +or- 20%

In the markscheme, the answer is 97 ohms + or - 10%

(Source: Question 8 from AS Level Physics A - H156/01 Breadth in Physics - 23 May 2017 - Morning)
Original post by vaishnoraj
Two resistors of resistances 120 ohms and 500 ohms are connected in parallel. The percentage uncertainty in the value of resistance of each resistor is 10%.

What is the correct value of the total resistance and the percentage uncertainty?


Original post by vaishnoraj
In the markscheme, the answer is 97 ohms + or - 10%

(Source: Question 8 from AS Level Physics A - H156/01 Breadth in Physics - 23 May 2017 - Morning)


To find the answer as 97 Ω ± 10%, do the following.

Find the maximum resistance Rmax and minimum resistance Rmin.

The error is

error=RmaxRmin2 error = \dfrac{R_{\max} - R_{\min}}{2}


The ‘correct” resistance can be calculated using 97 Ω and 120 Ω.
Reply 4
Sorry. I am not understanding.
Original post by vaishnoraj
Sorry. I am not understanding.


old_survivor had already shown how to obtain the effective resistance 97 Ω, which is I meant by “correct” resistance. I made a typo in post #4.

“The ‘correct” resistance can be calculated using 500 Ω and 120 Ω.”

Let assume R1 = 120 ± 10% and R2 = 500 ± 10%.

To obtain the maximum resistance, you use
R1 = 120 + 10% and R2 = 500 + 10%


1Rmax=1120+0.1×120+1500+0.1×500 \dfrac{1}{R_{\max}} = \dfrac{1}{120 + 0.1 \times 120} + \dfrac{1}{500 + 0.1 \times 500}


Similarly, you can find the minimum resistance by using
R1 = 120 10% and R2 = 500 10%


The uncertainty or error is then found by

error or uncertainty=RmaxRmin2 \text{error or uncertainty} = \dfrac{R_{\max} - R_{\min}}{2}


If you still don't understand, post what you have calculated or be specific of what you don't understand.

Quick Reply

Latest